10
$\begingroup$

Consider a uniform random permutation of $\{1,\dots, n\}$, and let $D_n$ be its number of descents (indices $i$ such that $\sigma(i)>\sigma(i+1)$). There is a nice result by Tanny where they show that $D_n$ has the same distribution as $\lfloor U_1 + \dots + U_n \rfloor$ where the $U_i$ are iid uniform in $[0,1]$.

Their proof is straightforward: write down the distributions explicitly, using Euler's formula for Eulerian numbers (the number of permutations with a fixed number of descents), and see that they are the same. But it looks like there should be a more probabilistic proof based on couplings between the two... Do you see one?

$\endgroup$

1 Answer 1

10
$\begingroup$

Yes. Put $V_i = U_1+U_2+\cdots+U_i$, $W_i = \lfloor V_i \rfloor$ and $X_i = V_i - W_i$.

I claim that the $X_i$ are independent and uniformly distributed in $[0,1]$. Indeed, conditional on $(U_1, \ldots, U_{i-1})$, we have $V_i = U_i + c$ for some constant $c$, so $V_i$ is (conditional on $(U_1, \ldots, U_{i-1})$) uniformly distributed in $[c, c+1]$, and then $X_i$ is uniform in $[0,1]$.

In particular, with probability $1$, the $X_i$ are distinct, so there is a unique order preserving map $\{ X_1, \ldots, X_n \} \longrightarrow \{ 1, 2, \ldots, n \}$. Let $\sigma$ be the permutation such that $X_i \mapsto \sigma(i)$. So $\sigma$ is uniform in $S_n$.

Then $\sigma(i) < \sigma(i+1)$ iff $W_i = W_{i-1}$, and $\sigma(i) > \sigma(i+1)$ iff $W_i = W_{i-1}+1$. So the total number of descents of $\sigma$ is $\sum_{i=1}^n (W_i - W_{i-1}) = W_n - W_0 = W_n$, as you requested.

This is a pretty standard construction. I think of it as coming from Stanley, "Eulerian partitions of a unit hypercube", 1977, although your citation is from 1973, so it was before Stanley!

$\endgroup$
1
  • 2
    $\begingroup$ Schmidt and Simion say that this result, or at least something very close, is "implicit in a result of Laplace" (see page 8). I think the relevant section of Laplace is here but I haven't actually deciphered what Laplace is saying. $\endgroup$ Commented Apr 18 at 15:57

You must log in to answer this question.

Start asking to get answers

Find the answer to your question by asking.

Ask question

Explore related questions

See similar questions with these tags.